Will give brainliest. Find the length and measure of each arc. Show your work.

Will Give Brainliest. Find The Length And Measure Of Each Arc. Show Your Work.

Answers

Answer 1

Answer:

exact arc length = 21pi

approximate arc length (using calculator) = 65.9734457253857

approximate arc length (using 3.14 for pi) = 65.94

units are in feet

============================================

Work Shown:

L = (x/360)*2*pi*r

L = (315/360)*2*pi*12

L = 21pi = exact arc length in terms of pi

If you use your calculator's stored version of pi, then L is approximately

L = 21pi

L = 65.9734457253857

Or if you end up using pi = 3.14, then

L = 21*pi

L = 21*3.14

L = 65.94

Fairly close to the other approximation though a bit off because pi = 3.14 is just a rough approximation.


Related Questions

P(x) =2x3 -11x2 -4x +1 g(x) =2x +1

Answers

Answer:

see explanation

Step-by-step explanation:

If (2x + 1) is a factor then x = - [tex]\frac{1}{2}[/tex] is a root and P(- [tex]\frac{1}{2}[/tex] ) = 0 ← Factor theorem

P(- [tex]\frac{1}{2}[/tex] )

= 2(- [tex]\frac{1}{2}[/tex] )³ - 11(- [tex]\frac{1}{2}[/tex] )² - 4(- [tex]\frac{1}{2}[/tex] ) + 1

= - [tex]\frac{1}{4}[/tex] - [tex]\frac{11}{4}[/tex] + 2 + 1

= - [tex]\frac{12}{4}[/tex] + 3

= - 3 + 3

= 0

Since P(- [tex]\frac{1}{2}[/tex] ) = 0 then g(x) is a factor of P(x)

a) Simplify the expression and explain each step. (2 points)


4(3x+2) -2
= ?

Answers

Answer:

6 (2 x + 1)

Step-by-step explanation:

Simplify the following:

4 (3 x + 2) - 2

Hint: | Distribute 4 over 3 x + 2.

4 (3 x + 2) = 12 x + 8:

12 x + 8 - 2

Hint: | Group like terms in 8 + 12 x - 2.

Grouping like terms, 8 + 12 x - 2 = 12 x + (8 - 2):

12 x + (8 - 2)

Hint: | Subtract 2 from 8.

8 - 2 = 6:

12 x + 6

Hint: | Factor out the greatest common divisor of the coefficients of 12 x + 6.

Factor 6 out of 12 x + 6:

Answer: 6 (2 x + 1)

-1/4 (x+2)+5=-x I really need help

Answers

Answer:

x = -6

Step-by-step explanation:

Answer:

-6

Step-by-step explanation:

-1/4 (x+2)+5=-x

-1/4x-1/2+5=-x

-1/4x+4.5=-x

+1/4x          +1/4x

(4/3) 4.5=-3/4x (4/3)

6=x

x=6

Mrs.magar sold 40 kg of fruits at rate of RS 60 per kg and gained Rs 600 . calculate purchasing rate and profit percent.​

Answers

Answer:

Purchasing rate = ₹45 per kg

Profit percent = 33.33%

Step-by-step explanation:

Selling price of 1 kg fruit = ₹60

Therefore, S. P. of 40 kg fruits = 40*60 = ₹2400

Gain = ₹ 600 (given)

Cost price (C. P.) = S. P. - Gain

= ₹2400 - ₹600

= ₹ 1800

Rate of purchasing

= C. P. /quantity of fruits

= 1800/40

= ₹45 per kg

Profit percent

= (Gain* 100)/C.P.

= (600*100)/1800

= 60000/1800

=33.33%

Help me you guys! it's important!​

Answers

Answer:

hey mate

surface area of cuboid is 2(lh+bh+hl)

doing for first cuboid

h = 20

l = 6

b = 5

surface area for this cuboid on calculating u get 500 sq.cm

now

second cuboid

h = 12

l = 6

b = 4

calculaye it by formula and u get 288 sq.cm

so total surface area is 500 + 288 = 788 but in this one side is getting counted 2 times which is rectangle with length 6 and 12 so it's area is 12×6 = 72

final ans = 788-72 = 716 cm2

as per me that's ans

brinliest pls

Can somebody please help me

Answers

Exact area = 25piApproximate area = 78.5

You can pick one or the other to have as your answer

===========================

Explanation:

The formula you use is

A = pi*r^2

The radius is r = d/2 = 10/2 = 5

So,

A = pi*r^2 = pi*5^2 = pi*25 = 25pi

is the exact area

Replace pi with 3.14 to get 25*pi = 25*3.14 = 78.5

which is the approximate area

To get a better approximation, you would use more decimal digits of pi.

Simplify 7.5 + n + 9.63.

Answers

Answer: Hi!

The only thing we're able to do to simplify this equation is combine our like terms. We have two like terms in the equation: 7.5 and 9.63.

7.5 + 9.63 = 17.13

Our equation now looks like this:

n + 17.13

We have nothing left to simplify, so we're done!

Hope this helps!

(b-2)x= 8
In the given equation, b is a constant. If the equation
has no solution, what is the value of b ? A) 2
B) 4
C) 6
D) 10

Answers

Answer:

2

Step-by-step explanation:

(2-2)x = 8

(0)x = 8

x = 8/0

no solution

The value of "b" that would result in the equation having no solution is A) 2.

To determine the value of "b" that would result in the given equation having no solution, we need to look at the coefficient of "x" in the equation (b - 2) and the constant term on the other side (8).

The equation is: (b - 2)x = 8

For the equation to have no solution, the coefficient of "x" (b - 2) must be 0. This is because when you multiply any number by 0, the result is always 0, meaning the left-hand side of the equation becomes 0x, which simplifies to 0. However, the right-hand side is 8, and 0 is never equal to 8.

Therefore, to make (b - 2) equal to 0, we can set:

b - 2 = 0

Adding 2 to both sides:

b = 2

So, the value of "b" that would result in the equation having no solution is A) 2.

To know more about equation"

https://brainly.com/question/29114761

#SPJ2

solve the equation:
|x+3| >2x-1

Answers

Answer:

x < 4

Step-by-step explanation:

|x+3| >2x-1 [|a| is the absolute value of a]

x+3 > 2x-1

4 > x

Therefore,

x < 4

The negative case doesn't have equal RHS and LHS so, it isn't valid.

find the supplement of 3/4 of a right angle

Answers

Answer:

Step-by-step explanation:

¾*90=67.5

180-67.5=112.5°

Answer:

60°

Step-by-step explanation:

You can see the explanation in the picture

evaluate : 8/-5+(4/-3)+1/3

Explain full steps
with easy method​

Answers

Answer:

-39/15

Step-by-step explanation:

=-8/5-4/3+1/3

Taking LCM of 5,3 and 3.

=3(-8)-5(4)+5(1)/15

=-24-20+5/15

=-44+5/15

=-39/15

Note:if you need to ask any question please let me know.

761.8 x 10^-8 Express the number in scientific notation. A) 7.618 x 10^-6 B) 7.618 x 10^-8 C) 7.618 x 10^2 D) 7.618 x 10^6

Answers

Answer:

[tex]\huge\boxed{A)\ 7.618\times10^{-6}}[/tex]

Step-by-step explanation:

The scientific notation:

[tex]a\cdot10^n[/tex]

where

[tex]1\leq a<10;\ n\in\mathbb{Z}[/tex]

We have

[tex]761.8\times10^{-8}[/tex]

We need to move the decimal point two places to the left.

[tex]\underbrace{(7.618\times10^2)}_{=761.8}\times10^{-8}=7.618\times(10^2\times10^{-8})[/tex]

use

[tex]a^n\cdot a^m=a^{n+m}[/tex]

[tex]=7.618\times10^{2+(-8)}=7.618\times10^{-6}[/tex]

Answer:

a

Step-by-step explanation:

The weight of a box varies directly as the volume of the box. If a 138-pound box has a volume of 23 gallons, what is the weight of a box that has a volume of 25 gallons? A. 31 pounds B. 150 pounds C. 138 pounds D. 29 pounds

Answers

Answer:

150 pounds

the weight is directly proportional to the volume. hence:

138/23 = 6

now, 25*6 = 150

Answer:

B. 150 pounds

Step-by-step explanation:

If LM = x + 3, MN = 2x and LN = 24, what does x equal

Answers

Answer:

[tex]\displaystyle 7 = x[/tex]

Step-by-step explanation:

[tex]\displaystyle 24 = [3 + x] + 2x → 24 = 3 + 3x → \frac{21}{3} = \frac{3x}{3} \\ \\ 7 = x[/tex]

I am joyous to assist you at any time.

A company ships coffee mugs using boxes in the shape of cubes. The function g(x) = gives the side length, in inches, for a cube with a volume of x cubic inches. Suppose the company decides to double the volume of the box. Which graph represents the new function?

Answers

Answer:

The graph is attached below.

Step-by-step explanation:

The volume of the box containing the coffee mugs is,

[tex]V=x^{3}[/tex]

Then the function representing the side length, in inches, for the box is:

[tex]g(x)=x[/tex]

Now, it is provided that the company decides to double the volume of the box.

That is, the new volume will be:

[tex]V_{n}=2x^{3}[/tex]

Then the side length, in inches, for the box will be:

[tex]g_{n}(x)=\sqrt[3]{2x^{3}} =\sqrt[3]{2}x[/tex]

Then the graph representing the function, formed using the following points is:

[tex]x\ \ \ \ \ \ \ \ \ g_{n}(x)\\\_\_\_\_\_\_\_\_\_\_\_\_\_\_\_\_\_\\0\ \ \ \ \ \ \ \ \ \ \ 0\\1\ \ \ \ \ \ \ \ \ \ \ 2^{1/3}[/tex]

Answer:

c

Step-by-step explanation:

Will Mark Brainlest helppp plss​

Answers

Answer:

you can ask from it will be easy for you actually I don't know answern

Which list shows the numbers 13.5, 1.35, 1.352, 0.135 in ascending order?
13.5, 1.352, 1.35, 0.135
0.135, 1.352, 1.35, 13.5
0.135, 13.5, 1.35, 1.352
0.135, 1.35, 1.352, 13.5

Answers

d ok have a good day

Answer: Choice D

0.135, 1.35, 1.352, 13.5

=====================================================

Explanation:

Ascending order means "smallest to largest"

The smallest item is 0.135, then we move to 1.35, then to 1.352, and finally 13.5

The reason why 1.35 is smaller than 1.352 is to think of the first value is 1.350; then note how 350 is smaller than 352.

find the center of a circle with the equation: x^2+y^2-32x-60y+1122=0

A. (24, 30)
B. (13, 20)
C. (14, 28)
D. (16, 30)

Answers

Answer:

-32x-60y+1122=0

-(32x+60y-1122)=0

32x+60y-1122=0

by factorize

Answer:

24,30

Step-by-step explanation:

help help help

help
help help

Answers

Answer:

R= (-9, -10) S=(-1,-10) T=(-1, -8) U=(-9, -8)

Step-by-step explanation:

There isn't really an explanation it's just reading the points on the graph. Hope this helps!! :)

Below are a list of costs and discounts for groceries. Round to the nearest dollar to estimate the total cost
+ $12.34
+ $5.07
- $0.73
+ $2.84
- $1.50

Answers

Answer:

$18

Step-by-step explanation:

$18.02 rounds to $18

Answer:

  $17

Step-by-step explanation:

Rounded, the listed numbers are ...

  12 + 5 -1 +3 -2 = 17

The estimate of total cost is $17.

Please help with this question

Answers

9514 1404 393

Answer:

  27.932 in

Step-by-step explanation:

The initial angle (or height) is not shown, so we have assumed it is 30°. The equation for the height of the valve cap can be written as a function of angle:

  y = 15.375 +14.5·sin(x +30) . . . . . . where x is in degrees

The angle measured from the +x axis is already 30° when the rotation angle is zero. Evaluating the above equation with x = 390° gives an angle of 420°, or 60° beyond one full rotation.

  y = 15.375 +14.5·sin(60°) ≈ 27.932 . . . . inches above the ground.

The valve cap is 27.932 in. above the ground.

Can someone help me with 1-5 and create problems for 6-10

Answers

The problem is that you need to subtract 15÷42 would get you 74 cherry pie will be a cantaloupe into a torch a computer is a wiring device that could help you in life the answer is 82

The altitude (i.e., height) of a triangle is increasing at a rate of 2 cm/minute while the area of the triangle is increasing at a rate of 1.5 square cm/minute. At what rate is the base of the triangle changing when the altitude is 12 centimeters and the area is 99 square centimeters

Answers

Answer:

A = 1/2 B H       area = 1/2 base X height

B = 2 A / H = 2 * 99 / 12 = 16.5 cm

dA / d t = 1/2 * (B dH / dt + H dB / dt)

dB / dt = (2 dA / dt - B dH / dt) / H

dB / dt = (2 * 1.5 cm^2 / min - 16.5 cm * 2 cm / min) / 12 cm

dB / dt = (3 - 33) / 12 cm/min = -2.5 cm/min

Evaluate Equation Below
Please help!

Answers

Answer:

2nd answer

Step-by-step explanation:

4x^3+2

4*(-1)^3+2

4*(-1)+2

-4+2

-2

Answer: -2

Step-by-step explanation:

Replacing x for -1 would make the equation [tex]4(-1^3)+2[/tex]. Knowing PEMDAS, we know that the parentheses goes first. [tex]-1^3[/tex] means that we need to multiply -1 by itself 3 times ([tex]-1*-1*-1[/tex],  * equals "times"). [tex]-1*-1=1[/tex] so [tex]1*-1=-1[/tex]. Now our equation is [tex]4(-1)+2[/tex].    [tex]4*-1=-1[/tex] and -4 + 2 = -2. Hope this explanation helps :)

Consider this polynomial, where a is an unknown real number.
p(t) = x^4 +5x^3 + ax^2 - 3x + 11
The remainder of the quotient of P(x), and (x+ 1) is 17.
Braulio uses synthetic division to find the value of a, and Zahra uses the remainder theorem to find the value of a.

Answers

Answer:

Brauilo is wrong because he divided by (x+1) instead of (x-1)

Step-by-step explanation:

The remainder Theorem states that

if polynomial f(x) is divided by binomial x-a, the remainder will equal f(a). The factor is positive so the binomial is the same as

which is why we divide by -1, or subsitue -1 into the equation p(x).

The remainder of a polynomial division can be gotten using remainder theorem or synthetic division.

The true statement is: Brauilo did not find the value of a, because he divided by (x+1) instead of (x-1)

From the question, we have the following parameters:

[tex]\mathbf{p(x) = x^4 +5x^3 + ax^2 - 3x + 11}[/tex]

[tex]\mathbf{Divisor =x+ 1}[/tex]

[tex]\mathbf{Remainder = 17}[/tex]

First, we set the divisor to 0.

[tex]\mathbf{Divisor =x+ 1 = 0}[/tex]

So, we have:

[tex]\mathbf{x+ 1 = 0}[/tex]

Solve for x

[tex]\mathbf{x= -1}[/tex]

The above equation means that, the value of x that will be used to test the polynomial is -1

From the question,

Zahra used [tex]\mathbf{x= -1}[/tex]; this is represented as: P(-1)Braulio used [tex]\mathbf{x= 1}[/tex]; this is represented in the synthetic division

Hence, Braulio is incorrect, because he used the wrong value of x

Read more about polynomial division at:

https://brainly.com/question/12011809

-5^2 find the power

Answers

Answer:

-25

Step-by-step explanation:

Answer:

Hey there!

When we have a^b, b is the power.

Thus, in -5^2, 2 is the power.

Let me know if this helps, or if you need more help :)

One thousandth of the base unit in length :

One millionth of the base unit in mass :

One thousandth of the base unite in time :

Answers

Step-by-step explanation:

[tex]1 \div 100th[/tex]

URGENT!!!!

The value of a12 is:

2
0
1
can't be done

Answers

it's 1 beacuse the exponent has no power so its 1

The correct answer from the given matrix shows that the value of a12 is 1

What is a Matrix?

This refers to a rectangular array of numbers that is arranged in rows and columns to indicate a mathematical property.

Hence, we can see that from the matrix given, we can see that there are different values for each one and to find a12, we can see that because there is no power in the exponent, the answer is 1.

Read more about matrixes here:

https://brainly.com/question/94574

#SPJ2

how many five card hands that contain all 4 aces can be dealt from a standard pack of 52 cards

Answers

Answer:

48

Step-by-step explanation:

52-4=48

hope this helps

WILL GIVE BRAINLIEST AND 20 POINTS! SHOW WORK!
How many different 6-digit phone numbers do not contain the digit 6? Assume that any digit in the phone number can be any of the remaining numbers. Use the Multiplication Principle of Counting to solve the problem.

Answers

Answer:

Step-by-step explanation:

We don't know if the number can start with a 0 or 1. Since we don't know, I will assume the number can start with either. The restriction is on 6s.

The way this is worded, I'm not sure if the digits can be repeated or not. It sounds like they can.

9 * 9 * 9 * 9 * 9 * 9 = 531441

If you are not allowed to repeat and no digit is a 6 then you get

9 *8 * 7 * 6 * 5*4 = 60480

Other Questions
7. Why did Virginia grow slowly? Find x in each triangle what number should replace the question mark In which type of speech is the speaker expected to express gratitude for receiving a specific honor, thank a few people who helped along the way, and should express knowledge about and appreciation of the qualities being celebrated Find the solution of the inequality 5 > r - 3.A) r Need a Blackout poem using the Declaration of Independence (hope for freedom) on a particular day, a man spent 12 minutes more driving to his office than driving home. His average speed from home to office is 12km/h and from office to home is 60m/h .How far is the man home to his office Help ASAP In a system of linear equations the two equations have the same slope. Describe the possible solutions to this equation. Judy's heart beats 70 times a minute. At this rate, how many times does it beat per hour? a regional manager for a pet supply chain, is responsible for keeping his employees updated on changes in diversity policies. Jared plays the role of a _______ in managing diversity. disseminator leader liaison figurehead communicator does x-2/x-6 simplify to 1/3 ?explain why or why not If v=5 and w=24, what is 5v+w? 19x - 45 = 240Plz solve for x Help Please Question 2Choose the correct answer based on the reading Un viaje interesante.Cmo puedes ver las lneas de Nazca?en autobsen taxien avinboth a and a What is 25x + 67y if x = 23 and y = 36. Give explanation please! which excerpt from the poem is an example of a transformation? is f(x)=sqrt{x}+3x an exponential function? the ratio of the temperature of room S to the temperature of room T is 2:3 and the ratio of the temperature of room T to the temperature of room U is 2:3 . Find the ratio of the temperature of room S to the temperature of room U why are cheerleaders important in any athletic program? In point forms Create a python program that display thisFactorial CalculatorEnter a positive integer: 55! = 1 x 2 x 3 x 4 x 5The factorial of 5 is: 120Enter a positive integer: 44! = 1 x 2 x 5 x 4.The factorial of 4 is: 24Enter a positive integer: -5Invalid input! Program stopped!